Jump to content

Photo

Đề thi Olympic toán sinh viên ĐH BK HN 2013


  • Please log in to reply
18 replies to this topic

#1
viet 1846

viet 1846

    Gà con

  • Thành viên
  • 224 posts
Đề thi Olympic toán sinh viên ĐH Bách Khoa Hà Nội

Môn: Giải tích

Câu 1: Tìm giới hạn: $\mathop {lim}\limits_{n \to \infty } \frac{{{1^5} + {2^5} + \cdots + {n^5}}}{{{n^6}}}$

Câu 2: Tìm $L = \mathop {lim}\limits_{m \to \infty } {\alpha _m}$ Với ${\alpha _m} = \mathop {Max}\limits_{x \in \left[ {0;1} \right]} \left( {x - {x^m}} \right)$

Câu 3: Cho hàm $u(x)$ dương liên tục trên $[0; \infty )$, hàm $\varphi \left( x \right)$ tăng và khả vi trên $[0;\infty )$, $\varphi \left( 0 \right)=1$

Biết rằng với mọi $x\ge 0$ ta có: $u\left( x \right) \le 1 + \int\limits_0^x {\frac{{\varphi '\left( t \right)}}{{\varphi \left( t \right)}}} u\left( t \right)dt$

Chứng minh: $u(x) \le \varphi \left( x \right)$ trên $[0;\infty )$


Câu 4: Cho $f_1(x)=4x^3-3x$ , $f_{n+1}=f_1(f_n)$

Tính: $\mathop {lim}\limits_{m \to \infty } \int\limits_{ - 1}^1 {f_n^2} \left( x \right)dx$


Câu 5: Tìm tất cả hàm $f(x)$ xác định trên $(0;\infty )$ Và khả vi 2 lần thỏa mãn :

$\left\{ \begin{array}{l} f'\left( x \right) > 0\\ f\left( {f'\left( x \right)} \right) = - f(x) \end{array} \right.$


Edited by E. Galois, 31-01-2013 - 18:45.


#2
viet 1846

viet 1846

    Gà con

  • Thành viên
  • 224 posts
Các mod có gì sử hộ em với sữa mãi không được: :(

Đi thi làm được 3 bài. :(

Bài 1: Câu này khá đơn giản:

\[\mathop {lim}\limits_{n \to \infty } \frac{{{1^5} + {2^5} + \cdots + {n^5}}}{{{n^6}}} = \mathop {lim}\limits_{n \to \infty } \frac{1}{n}{\sum\limits_1^n {\left( {\frac{i}{n}} \right)} ^5} = \int\limits_0^1 {{x^5}} dx = \frac{1}{6}\]

Bài 2:

Xét hàm số:

$f(x)=x-x^m$ trên $[0;1]$

\[f'(x) = 1 - m{x^{m - 1}} = 0 \Leftrightarrow x = \frac{1}{{{m^{\frac{1}{{m - 1}}}}}} = {m^{\frac{1}{{1 - m}}}} \in (0;1)\,\forall m \ge 2\]

Ta có $f'$ đổi dấu từ dương sang âm khi $x$ qua ${m^{\frac{1}{{1 - m}}}}$

Vậy \[{\alpha _m} = \mathop {Max}\limits_{x \in \left[ {0;1} \right]} \left( {x - {x^m}} \right) = f\left( {{m^{\frac{1}{{1 - m}}}}} \right) = {m^{\frac{1}{{1 - m}}}}\left( {1 - \frac{1}{m}} \right)\]

Ta có:

\[\mathop {lim}\limits_{m \to \infty } {m^{\frac{1}{{1 - m}}}} = \mathop {lim}\limits_{m \to \infty } {e^{\frac{{\ln m}}{{1 - m}}}}{\mkern 1mu} {\mkern 1mu} = {e^{\mathop {lim}\limits_{m \to \infty } \frac{{\ln m}}{{1 - m}}}} = {e^{\mathop {lim}\limits_{m \to \infty } \frac{{ - 1}}{m}}} = 1\]

Nên \[\mathop {lim}\limits_{m \to \infty } {\alpha _m} = 1\]

Bài 4: Đặt $x=cost$ suy ra $f_1(x)=cos3t$.......$f_n(x)=cos(3^n.t)$

Ta có: $dx=-sintdt$

$x=-1 \to t=\pi$

$x=1 \to t=0$

Ta có

\[{I_n} = \int\limits_{ - 1}^1 {f_n^2} \left( x \right)dx = \int\limits_0^\pi {\sin t.{{\left[ {cos\left( {{3^n}t} \right)} \right]}^2}} dt = \frac{1}{2}\int\limits_0^\pi {\sin t.\left[ {1 + cos\left( {{{2.3}^n}t} \right)} \right]} dt\]


\[{I_n} = \frac{1}{2}\int\limits_0^\pi {\sin tdt} + \frac{1}{4}\int\limits_0^\pi {\left[ {\sin \left( {1 + {{2.3}^n}} \right)t + \sin \left( {1 - {{2.3}^n}} \right)t} \right]} dt\]


\[{I_n} = 1 - \frac{1}{4}\left[ {\frac{{cos\left( {1 + {{2.3}^n}} \right)\pi - 1}}{{1 + {{2.3}^n}}} + \frac{{cos\left( {1 - {{2.3}^n}} \right)\pi - 1}}{{1 - {{2.3}^n}}}} \right]\]

Suy ra:

\[\mathop {lim}\limits_{n \to \infty } {I_n} = 1\]

--------------------------------------------------------------------------------------------------------

hic, lúc thi thay cận là $0\to 1$ mất rồi điên mất

Edited by E. Galois, 26-01-2013 - 12:57.


#3
vo van duc

vo van duc

    Thiếu úy

  • ĐHV Toán Cao cấp
  • 582 posts
Có đề Đại số không anh em? hi

Võ Văn Đức 17.gif       6.gif

 

 

 

 

 


#4
viet 1846

viet 1846

    Gà con

  • Thành viên
  • 224 posts
Em không thi đại số anh ak. :D

#5
dangnamneu

dangnamneu

    Hạ sĩ

  • Thành viên
  • 68 posts
Nhận xét: Đề thi cũng khá đơn giản, không khó nhiều câu quen thuộc.
Mình xin được góp vui Câu 3:
Thực chất đây chính là dạng của bất đẳng thức Gronwall. Ta có chứng minh như sau:
Theo điều kiện bài toán ta có:

$\frac{{u(x)}}{{1 + \int\limits_0^x {\frac{{\varphi '(t)}}{{\varphi (t)}}u(t)dt} }} \le 1,\forall x \in \left[ {0, + \infty } \right)$.

Suy ra

$\left[ {\ln \left( {1 + \int\limits_0^x {\frac{{\varphi '(t)}}{{\varphi (t)}}u(t)dt} } \right)} \right]' \le \frac{{\varphi '(x)}}{{\varphi (x)}} = \left[ {\ln \left( {\varphi (x)} \right)} \right]'$.


Lấy tích phân hai vế trên $\left[ {0,x} \right]$ của bất đẳng thức trên ta được:


$\int\limits_0^x {\left[ {\ln \left( {1 + \int\limits_0^x {\frac{{\varphi '(t)}}{{\varphi (t)}}u(t)dt} } \right)} \right]'dx \le \int\limits_0^x {\left[ {\ln \left( {\varphi (x)} \right)} \right]'dx} } \Rightarrow \ln \left( {1 + \int\limits_0^x {\frac{{\varphi '(t)}}{{\varphi (t)}}u(t)dt} } \right) \le \ln \left( {\varphi (x)} \right)$.


Suy ra
$1 + \int\limits_0^x {\frac{{\varphi '(t)}}{{\varphi (t)}}u(t)dt} \le \varphi (x) \Rightarrow u(x) \le 1 + \int\limits_0^x {\frac{{\varphi '(t)}}{{\varphi (t)}}u(t)dt} \le \varphi (x)$.


Bài toán được chứng minh.

Giáo viên môn Toán tại website : http://vted.vn


#6
dangnamneu

dangnamneu

    Hạ sĩ

  • Thành viên
  • 68 posts

Bài 4: Đặt $x=cost$ suy ra $f_1(x)=cos3t$.......$f_n(x)=cos(3^n.t)$

Ta có: $dx=-sintdt$

$x=-1 \to t=\pi$

$x=1 \to t=0$

Ta có

\[{I_n} = \int\limits_{ - 1}^1 {f_n^2} \left( x \right)dx = \int\limits_0^\pi {\sin t.{{\left[ {cos\left( {{3^n}t} \right)} \right]}^2}} dt = \frac{1}{2}\int\limits_0^\pi {\sin t.\left[ {1 + cos\left( {{{2.3}^n}t} \right)} \right]} dt\]


\[{I_n} = \frac{1}{2}\int\limits_0^\pi {\sin tdt} + \frac{1}{4}\int\limits_0^\pi {\left[ {\sin \left( {1 + {{2.3}^n}} \right)t + \sin \left( {1 - {{2.3}^n}} \right)t} \right]} dt\]


\[{I_n} = 1 - \frac{1}{4}\left[ {\frac{{cos\left( {1 + {{2.3}^n}} \right)\pi - 1}}{{1 + {{2.3}^n}}} + \frac{{cos\left( {1 - {{2.3}^n}} \right)\pi - 1}}{{1 - {{2.3}^n}}}} \right]\]

Suy ra:

\[\mathop {lim}\limits_{n \to \infty } {I_n} = 1\]

--------------------------------------------------------------------------------------------------------

hic, lúc thi thay cận là $0\to 1$ mất rồi điên mất


Chỉ đơn giản như sau bạn à:


$\int\limits_{ - 1}^1 {f_n^2(t)dt} = \int\limits_{ - 1}^1 {c{\rm{o}}{{\rm{s}}^2}\left( {{3^n}t} \right)dt} = 1 + \frac{1}{2}\int\limits_{ - 1}^1 {c{\rm{os}}\left( {{{2.3}^n}t} \right)dt} = 1 + \frac{1}{{{{2.3}^n}}}\sin \left( {{{2.3}^n}} \right) \to 1$


Edited by phudinhgioihan, 27-01-2013 - 15:09.

Giáo viên môn Toán tại website : http://vted.vn


#7
viet 1846

viet 1846

    Gà con

  • Thành viên
  • 224 posts

Chỉ đơn giản như sau bạn à:


$\int\limits_{ - 1}^1 {f_n^2(t)dt} = \int\limits_{ - 1}^1 {c{\rm{o}}{{\rm{s}}^2}\left( {{3^n}t} \right)dt} = 1 + \frac{1}{2}\int\limits_{ - 1}^1 {c{\rm{os}}\left( {{{2.3}^n}t} \right)dt} = 1 + \frac{1}{{{{2.3}^n}}}\sin \left( {{{2.3}^n}} \right) \to 1$


Mình nghĩ là bài bạn có vấn đề, việc thay $x=cost$ thì ta cũng cần phải đổi vi phân chứ.

#8
ssupermeo

ssupermeo

    Binh nhì

  • Thành viên
  • 13 posts

Bài 1. cho MT $A=\begin{pmatrix} -1 &3 \\ -1&2 \end{pmatrix}$ và $B=\begin{pmatrix} \frac{\sqrt{3}-1}{2} &-1 \\ \frac{1}{2}& \frac{\sqrt{3}+1}{2} \end{pmatrix}$

a. Cmr $A^2-A+E=0$ tính $f(A)=E+\sum_{2013}^{k=1}(-1)^kA^k$ với E là ma trận đơn vị cấp 2

b. Tính $B^{2016}$



Bài 2. Cho ma trận $A$ là một ma trận thực, vuông cấp $n$. CMR $det(A-A^t)\geq 0$ với $A^t$ là ma trận chuyển vị của ma trận A



Bài 3. Cho ma trận A là ma trận vuông cấp n. Vết của $A$, kí hiệu $tr(A)$ là tổng các phần tử chéo của $A$. Ma trận $A$ gọi là ma trận lũy đẳng nếu $A^2=A$. CMR:

a. Nếu $A$ là ma trận lũy đẳng thì $A$ chéo hóa được

b. $A$ là ma trận lũy đẳng khi và chỉ khi $rank(A)=tr(A)$ và $rank(E-A)= tr(E-A)$



Bài 4. Tính định thức của ma trận vuông cấp 2013 $A=[a_{ij}]$ với:
$a_{ij}=\left\{\begin{matrix} b & khi & i< j\\ a & khi &i=j \\ -b & khi & i> j \end{matrix}\right.$



Bài 5. Cho đa thức $f(x)\in R[x]$ có ít nhất 2 nghiệm thực.


CMR đa thức $p(x)=f(x)-4026f'(x)+2013f''(x)$ cũng có ít nhất 2 nghiệm


Edited by phudinhgioihan, 31-01-2013 - 10:26.


#9
vo van duc

vo van duc

    Thiếu úy

  • ĐHV Toán Cao cấp
  • 582 posts
Câu 1.a)

Dùng định lý Caley-Haminton hay tính toán trực tiếp ta có đẳng thức $A^{2}-A+E=O$

Ta có:

$f(A)=E-A+A^{2}- A^{3} + A^{4}- A^{5}+...+ A^{2012}- A^{2013}$

$=(E-A)+(E-A). A^{2}+ (E-A). A^{4}+...+ (E-A). A^{2012}$

$=(E-A).( E+A^{2}+ A^{4}+...+ A^{2012})$

Ta dễ dàng chứng minh được rằng

$A^{6k}=E$

$A^{6k+1}=A$

$A^{6k+2}=A-E$

$A^{6k+3}=-E$

$A^{6k+4}=-A$

$A^{6k+5}=E-A$

Suy ra:

$A^{2}+ A^{4}+ A^{6}=O$

$A^{8}+ A^{10}+ A^{12}=O$

...

$A^{2008}+ A^{2010}+ A^{2012}=O$

Vậy $f(A)=E-A$

..............
Online bằng di động nên không định dạng cho đẹp được. Anh em thông cảm nha!

Edited by vo van duc, 30-01-2013 - 15:44.

Võ Văn Đức 17.gif       6.gif

 

 

 

 

 


#10
vo van duc

vo van duc

    Thiếu úy

  • ĐHV Toán Cao cấp
  • 582 posts
Câu 1.b)

Ta phân tích

$A=\begin{pmatrix} \frac{\sqrt{3}-1}{2} & -1 \\ \frac{1}{2} & \frac{\sqrt{3}+1}{2} \end{pmatrix}$

$=\begin{pmatrix} \cos \frac{\pi}{6}-\sin \frac{\pi}{6} & -2 \sin \frac{\pi}{6} \\ \sin \frac{\pi}{6} & \cos \frac{\pi}{6}+\sin \frac{\pi}{6} \end{pmatrix}$

Chứng minh quy nạp ta có

$A^{n}=\begin{pmatrix} \cos n\frac{\pi}{6}-\sin n\frac{\pi}{6} & -2 \sin n\frac{\pi}{6} \\ \sin n\frac{\pi}{6} & \cos n\frac{\pi}{6}+\sin n\frac{\pi}{6} \end{pmatrix}$

Với $n=2016$ thì $A^{2016}=\begin{pmatrix} 1 & 0 \\ 0 & 1 \end{pmatrix}$

Võ Văn Đức 17.gif       6.gif

 

 

 

 

 


#11
vo van duc

vo van duc

    Thiếu úy

  • ĐHV Toán Cao cấp
  • 582 posts
Câu 4:

Xét định thức cấp n

$D_{n}=\begin{vmatrix} -b & -b & -b & \cdots & -b & -b \\ a & -b & -b & \cdots & -b & -b \\ b & a & -b & \cdots & -b & -b \\ \vdots & \vdots & \vdots & \ddots & \vdots & \vdots \\ b & b & b & \cdots & -b & -b \\ b & b & b & \cdots & a & b \end{vmatrix}$

Thay cột 1 bằng cột 1 cộng cột n, ta có

$D_{n}=\begin{vmatrix} a-b & -b & -b & \cdots & -b & -b \\ 0 & -b & -b & \cdots & -b & -b \\ 0 & a & -b & \cdots & -b & -b \\ \vdots & \vdots & \vdots & \ddots & \vdots & \vdots \\ 0 & b & b & \cdots & -b & -b \\ a+b & b & b & \cdots & a & b \end{vmatrix}$

$=(a-b).D_{n-1} + (a+b).(-1)^{n+1}.D$

Với $D=\begin{vmatrix} -b & -b & -b & \cdots & -b & -b \\ a & -b & -b & \cdots & -b & -b \\ b & a & -b & \cdots & -b & -b \\ \vdots & \vdots & \vdots & \ddots & \vdots & \vdots \\ b & b & b & \cdots & -b & -b \\ b & b & b & \cdots & a & -b \end{vmatrix}_{n-1}$

$=\begin{vmatrix} -b & -b & -b & \cdots & -b & -b \\ a+b & 0 & 0 & \cdots & 0 & 0 \\ 2b & a+b & 0 & \cdots & 0 & 0 \\ \vdots & \vdots & \vdots & \ddots & \vdots & \vdots \\ 2b & 2b & 2b & \cdots & 0 & 0 \\ 2b & 2b & 2b & \cdots & a+b & 0 \end{vmatrix}_{n-1}$

$=(-b).(-1)^{n}.(a+b)^{n-2}$

Vậy $D_{n}=(a-b).D_{n-1}+b.(a+b)^{n-1}$

Ta có:

$D_{1}=a=\frac{1}{2}.\left [ (a-b)^{1} + (a+b)^{1} \right ]$

$D_{2}=a^{2}+b^{2}=\frac{1}{2}.\left [ (a-b)^{2} + (a+b)^{2} \right ]$

Dự đoán $D_{n}=\frac{1}{2}.\left [ (a-b)^{n} + (a+b)^{n} \right ] (*)$

Thật vậy. Ta chứng minh $(*)$ bằng quy nạp.

Giả sử $(*)$ đúng với $n=k$, tức là $D_{k}=\frac{1}{2}.\left [ (a-b)^{k} + (a+b)^{k} \right ]$

Ta có

$D_{k+1}=(a-b).D_{k}+b.(a+b)^{k}$

$=(a-b).\frac{1}{2}.\left [ (a-b)^{k}+(a+b)^{k} \right ] +b.(a+b)^{k}$

$=\frac{1}{2}.(a-b)^{k+1}+ \frac{1}{2}.(a-b).(a+b)^{k}+ b.(a+b)^{k}$

$=\frac{1}{2}.(a-b)^{k+1}+\left [ \frac{1}{2}.(a-b)+b \right ].(a+b)^{k}$

$=\frac{1}{2}.\left [ (a-b)^{k+1} + (a+b)^{k+1} \right ]$

Vậy $(*)$ đúnh với $n=k+1$.
Theo nguyên lý quy nạp thì ta chứng minh được

$D_{n}=\frac{1}{2}.\left [ (a-b)^{n} + (a+b)^{n} \right ]$


Thay $n=2013$ ta có đáp số của đề.

..........................
Soạn trên dđ nên chắc có sai sót. phudinhgioihan sửa lại giúp anh với nha! hi

Edited by vo van duc, 30-01-2013 - 09:52.

Võ Văn Đức 17.gif       6.gif

 

 

 

 

 


#12
vo van duc

vo van duc

    Thiếu úy

  • ĐHV Toán Cao cấp
  • 582 posts
Câu 2:

Nhận xét rằng ma trận $A-A^{t}$ là ma trận phản đối xứng. Đang nghiên cứu về các tính chất về định thức, giá trị riêng của ma trận phản đối xứng để giải quyết bài này. Chỉ là ý tưởng thôi.

Anh em giúp đở một số tính chất của ma trận phản đối xứng đi. Một ma trận đẹp. hi

Võ Văn Đức 17.gif       6.gif

 

 

 

 

 


#13
GreatLuke

GreatLuke

    Binh nhất

  • Thành viên
  • 46 posts
Câu 2 Đại số:

Ma trận phản đối xứng $M$ nếu có 1 giá trị riêng là $x$ thì cũng có giá trị riêng là $-x$.

Nếu nó có bậc lẻ thì phải có ít nhất 1 giá trị riêng bằng $0$ nên có $detM=0$.

Nếu bậc của ma trận là 1 số chẵn:

Chứng minh (bằng phản chứng) ma trận phản đối xứng $M$ nếu có 1 giá trị riêng x khác $0$ thì đó là số thuần ảo.

Mặt khác $-x$ cũng là 1 giá trị riêng của M.

$detM$ bằng tích các giá trị riêng nên đi đến kết luận $detM \geq 0$.

Edited by GreatLuke, 30-01-2013 - 23:38.


#14
GreatLuke

GreatLuke

    Binh nhất

  • Thành viên
  • 46 posts

Câu 1.a)

Dùng định lý Caley-Haminton hay tính toán trực tiếp ta có đẳng thức $A^{2}-A+E=O$

Ta có:

$f(A)=E-A+A^{2}- A^{3} + A^{4}- A^{5}+...+ A^{2}- A^{2}$

$=(E-A)+(E-A). A^{2}+ (E-A). A^{4}+...+ (E-A). A^{2012}$

$=(E-A).( E+A^{2}+ A^{4}+...+ A^{2012})$

Ta dễ dàng chứng minh được rằng

$A^{6k}=E$

$A^{6k+1}=A$

$A^{6k+2}=A-E$

$A^{6k+3}=-E$

$A^{6k+4}=-A$

$A^{6k+5}=E-A$

Suy ra:

$A^{2}+ A^{4}+ A^{6}=O$

$A^{8}+ A^{10}+ A^{12}=O$

...

$A^{2008}+ A^{2010}+ A^{2012}=O$

Vậy $f(A)=E-A$

..............
Online bằng di động nên không định dạng cho đẹp được. Anh em thông cảm nha!

Câu này em tính ra là $f(A)=E$ cơ. mà nhóm cũng đơn giản chứ không dài như anh

#15
GreatLuke

GreatLuke

    Binh nhất

  • Thành viên
  • 46 posts

3. Cho ma trận A là ma trận vuông cấp n. Vết của A, kí hiệu tr(A) là tổng các phần tử chéo của A. Ma trận A gọi là ma trận lũy đẳng nếu $A^2=A$. CMR:

a. Nếu A là ma trận lũy đẳng thì A chéo hóa được

b. A là ma trận lũy đẳng khi và chỉ khi $rank(A)=tr(A)$ và $rank(E-A)= tr(E-A)$

a. Đa thức triệt tiêu của $A$ tách đơn nên $A$ chéo hóa được.

b.
Thuận:

$A$ lũy đẳng nên $A$ chéo hóa được. Các giá trị riêng của $A$ là $0$ hoặc $1$

Khi đó $rankA=n-dim(Ker(A))=n-dim(KGCR(A,0))=dim(KGCR(A,1))=tr(A)$

Dễ dàng chứng minh được $rank(E-A)=tr(E-A)$(có thể sử dụng dạng chéo hóa của ma trận $A$).

Đảo:

Vì $tr(A)=rank(A)$ và $tr(E-A)=rank(E-A)$ nên $rank(A)+rank(E-A)=tr(A)+tr(E-A)=n$.

Chứng minh đẳng thức sau: $rank(A)+rank(E-A)=n+rank((E-A)A)$.
Trước hết có bất đẳng thức $rank(A)+rank(B)\leq n+rank(AB)$.

Mặt khác ta có:
Nếu $X_{1}$ thuộc $Ker(A)$, $X_{2}$ thuộc $Ker(E-A)$ . Xét $X_{1}+X_{2}=0$.
Khi đó $A(X_{1}+X_{2})=0$ nên $X_{2}=0$(do $AX_{1}=0$;$AX_{2}=X_{2}$) và $X_{1}=0$. Vậy $Ker(A)$ và $Ker(E-A)$ có tổng trực tiếp.

Với mọi $Y$ thuộc $Ker(A)+Ker(E-A)$ thì $Y$ thuộc $Ker(A(E-A))$. Vậy $dim(Ker(A)+Ker(E-A))=dim(Ker(A))+dim(Ker(E-A))\leq dim(Ker(A(E-A))$. Vì $dim(Ker(A))+rank(A)=n$ nên $rank(A)+rank(E-A)\geq n+rank(A(E-A))$.

Kết hợp lại ta có $rank(A)+rank(E-A)=n+rank((E-A)A)=n$.

Vậy $rank(A-A^{2})=0$ hay $A$ lũy đẳng

p/s:Mình tự thấy cách của mình ko hay. Bạn nào có cách khác thì post lên cho mình học tập.

Edited by GreatLuke, 30-01-2013 - 17:50.


#16
phudinhgioihan

phudinhgioihan

    PĐGH$\Leftrightarrow$TDST

  • Biên tập viên
  • 348 posts


Bài 5. Cho đa thức $f(x)\in R[x]$ có ít nhất 2 nghiệm thực.


CMR đa thức $p(x)=f(x)-4026f'(x)+2013f''(x)$ cũng có ít nhất 2 nghiệm


Nếu $f$ bậc chẵn thì 2 bài tổng quát mình đã giải ở đây:

http://diendantoanho...-p/#entry249

Nếu đa thức $f$ bậc lẻ ( $\deg (f)=2n+1 \;, n \in \mathbb{N}$), khi đó, tồn tại $2 $ số thực $a_i$ sao cho

$$f(x)=(x-a_1)(x-a_2) H_{2n-1}(x)$$

Với $H_{2n-1}$ là đa thức bậc $2n-1$ và do đó bậc lẻ nên có ít nhất một nghiệm thực, do đó $f$ có ít nhất $3$ nghiệm thực (kể cả bội).


Với $t_1 \neq 0$

Xét $q(x)=e^{\frac{x}{t_1}} f(x)$

Do $f$ có ít nhất 3 nghiệm thực nên $q(x)$ cũng có ít nhất 3 nghiệm thực.

$$q'(x)=e^{\frac{x}{t_1}} \dfrac{1}{t_1}(f(x)+t_1f'(x))$$

suy ra $q'(x)=0$ có ít nhất 2 nghiệm thực, do đó $r(x)=f(x)+t_1f'(x)$ có ít nhất 2 nghiệm thực. Do $\deg( r )=\deg(f)$ nên $r$ có ít nhất 3 nghiệm thực.

Tương tự, với $t_2 \neq 0$ ta cũng có $h(x)=r(x)+t_2r'(x)=f(x)+(t_1+t_2)f'(x)+t_1t_2f''(x)$ có ít nhất 3 nghiệm thực.

Chọn $t_1+t_2=-4026 \;, t_1t_2=2013$, do phương trình $x^2-4026x+2013=0$ có 2 nghiệm thực nên tồn tại cắp số thực $t_1,t_2$ như thế. Vây ta có

$p(x)=f(x)-4026f'(x)+2013f''(x)$ có ít nhất 3 nghiệm thực.

Phủ định của giới hạn Posted Image

Đó duy sáng tạo ! Posted Image


https://phudinhgioihan.wordpress.com/

#17
LangTu Mua Bui

LangTu Mua Bui

    Binh nhất

  • Thành viên
  • 43 posts
Câu 5 
$\left\{ \begin{array}{l} f'\left( x \right) > 0\\ f\left( {f'\left( x \right)} \right) = - f(x) \end{array} \right.$

Ta có $f(f'(x))=-f(x)$

Thay $ x=f'(x) \Rightarrow f(f'(f'(x))))=f(x)$ Do $f'(x)>0 \Rightarrow f'(f'(x))=x  (1)$

$ \Rightarrow  f(f'(x))=-f(x)$

Do hàm khả vi cấp 2 nên Ta  đạo hàm 2 vế theo x $ \Rightarrow f''(x)f'(f'(x))=-f'(x)(2)$
 
Từ $ (1)(2)f''(x).f'(f'(x))\Leftrightarrow f''(x)x+f'(x)=0$

$ \Rightarrow f(x)=a\ln{x}+b$ Thay ngược lại đề bài $\Rightarrow  a=b=1 \Rightarrow f(x)=\ln{x} $

Edited by LangTu Mua Bui, 05-12-2015 - 12:03.


#18
LangTu Mua Bui

LangTu Mua Bui

    Binh nhất

  • Thành viên
  • 43 posts
 Cách khác cho câu 3

$u(x)\leq 1+\int_{0}^{x}\frac{\varphi '(t)u(t)dt}{\varphi (t)}$

Dễ thấy $ u(0) \leq 1$
 
$\Leftrightarrow u(x)-\varphi (x)  \leq  \int_{0}^{x} \frac{\varphi '(t)u(t)dt}{\varphi (t)}-\int_{0}^{x}\varphi '(t)dt=\int_{0}^{x}\left (\varphi '(t)( \frac{u(t)-\varphi (t)}{\varphi (t)})  \right ) dt $

Do $\varphi (t)$ đồng biến và $\varphi(0)=1 \Rightarrow \varphi (t)\geq 1 \forall t\in [0;\infty) $

$\Rightarrow u(x)-\varphi (x) \leq  \int_{0}^{x}\left (\varphi '(t)(u(x)-1)  \right ) dt=u(x)-\varphi (x)-\int_{0}^{x}\varphi '(x) $

$\Rightarrow \varphi(x)<1-\int_{0}^{x}u'(t)\varphi (t)dt ;\varphi(x)\geq 1 \forall x\in [0;\infty] \Rightarrow  u'(t)<0 $

Xét hàm số $g(x)=u(x)-\varphi (x) $

$g'(x)=u'(x)-\varphi' (x) <0 ;g(0)=u(0)-\varphi (0)<0 \Rightarrow g(x)<0 \forall x\in [0;\infty]$


#19
luuvanthai

luuvanthai

    Sĩ quan

  • Thành viên
  • 373 posts

 

 Cách khác cho câu 3

$u(x)\leq 1+\int_{0}^{x}\frac{\varphi '(t)u(t)dt}{\varphi (t)}$

Dễ thấy $ u(0) \leq 1$
 
$\Leftrightarrow u(x)-\varphi (x)  \leq  \int_{0}^{x} \frac{\varphi '(t)u(t)dt}{\varphi (t)}-\int_{0}^{x}\varphi '(t)dt=\int_{0}^{x}\left (\varphi '(t)( \frac{u(t)-\varphi (t)}{\varphi (t)})  \right ) dt $

Do $\varphi (t)$ đồng biến và $\varphi(0)=1 \Rightarrow \varphi (t)\geq 1 \forall t\in [0;\infty) $

$\Rightarrow u(x)-\varphi (x) \leq  \int_{0}^{x}\left (\varphi '(t)(u(x)-1)  \right ) dt=u(x)-\varphi (x)-\int_{0}^{x}\varphi '(x) $

$\Rightarrow \varphi(x)<1-\int_{0}^{x}u'(t)\varphi (t)dt ;\varphi(x)\geq 1 \forall x\in [0;\infty] \Rightarrow  u'(t)<0 $

Xét hàm số $g(x)=u(x)-\varphi (x) $

$g'(x)=u'(x)-\varphi' (x) <0 ;g(0)=u(0)-\varphi (0)<0 \Rightarrow g(x)<0 \forall x\in [0;\infty]$

 

Dòng này có vấn đề??






1 user(s) are reading this topic

0 members, 1 guests, 0 anonymous users